Can we prove that $f'(x)<2f(x)$ for all $x$ using the given conditions?

  • MHB
  • Thread starter Ackbach
  • Start date
  • Tags
    2017
In summary, to prove that $f'(x)<2f(x)$ for all $x$, the given conditions are that $f$ is a differentiable function and $f'(x)=2f(x)$ for at least one value of $x$. This can be proven using the Mean Value Theorem or proof by contradiction. The same proof can be applied for any value greater than 2. Without the given conditions, it is not possible to prove that $f'(x)<2f(x)$ for all $x$, but there may be other methods or approaches that can be used.
  • #1
Ackbach
Gold Member
MHB
4,155
89
Here is this week's POTW:

-----

Let $f$ be a real function with a continuous third derivative such that $f(x),f'(x), f''(x), f'''(x)$ are positive for all $x$. Suppose that $f'''(x)\leq f(x)$ for all $x$. Show that $f'(x)<2f(x)$ for all $x$.

-----

Remember to read the http://www.mathhelpboards.com/showthread.php?772-Problem-of-the-Week-%28POTW%29-Procedure-and-Guidelines to find out how to http://www.mathhelpboards.com/forms.php?do=form&fid=2!
 
Physics news on Phys.org
  • #2
Re: Problem Of The Week # 257 - Mar 25, 2017

This was Problem B-4 in the 1999 William Lowell Putnam Mathematical Competition.

No one answered this week's POTW. The solution, attributed to Kiran Kedlaya and his associates, follows:

(based on work by Daniel Stronger) We make repeated use of the following fact: if $f$ is a differentiable function on all of $\mathbb{R}$, $\lim_{x \to -\infty} f(x) \geq 0$, and $f'(x) > 0$ for all $x \in \mathbb{R}$, then $f(x) > 0$ for all $x \in \mathbb{R}$. (Proof: if $f(y) < 0$ for some $x$, then $f(x)< f(y)$ for all $x<y$ since $f'>0$, but then $\lim_{x \to -\infty} f(x) \leq f(y) < 0$.)

From the inequality $f'''(x) \leq f(x)$ we obtain
\[
f'' f'''(x) \leq f''(x) f(x) < f''(x) f(x) + f'(x)^2
\]
since $f'(x)$ is positive. Applying the fact to the difference between the right and left sides, we get
\begin{equation}
(1) \qquad \qquad \frac{1}{2} (f''(x))^2 < f(x) f'(x).
\end{equation}

On the other hand, since $f(x)$ and $f'''(x)$ are both positive for all $x$, we have
\[
2f'(x) f''(x) < 2f'(x)f''(x) + 2f(x) f'''(x).
\]
Applying the fact to the difference between the sides yields
\begin{equation}
(2) \qquad \qquad f'(x)^2 \leq 2f(x) f''(x).
\end{equation}
Combining (1) and (2), we obtain
\begin{align*}
\frac{1}{2} \left( \frac{f'(x)^2}{2f(x)} \right)^2
&< \frac{1}{2} (f''(x))^2 \\
&< f(x) f'(x),
\end{align*}
or $(f'(x))^3 < 8 f(x)^3$. We conclude $f'(x) < 2f(x)$, as desired.

Note: one can actually prove the result with a smaller constant in place of 2, as follows. Adding $\frac{1}{2} f'(x) f'''(x)$ to both sides of (1) and again invoking the original bound $f'''(x) \leq f(x)$, we get
\begin{align*}
\frac{1}{2} [f'(x) f'''(x) + (f''(x))^2] &< f(x) f'(x) + \frac{1}{2} f'(x) f'''(x) \\
&\leq \frac{3}{2} f(x) f'(x).
\end{align*}
Applying the fact again, we get
\[
\frac{1}{2} f'(x) f''(x) < \frac{3}{4} f(x)^2.
\]
Multiplying both sides by $f'(x)$ and applying the fact once more, we get
\[
\frac{1}{6} (f'(x))^3 < \frac{1}{4} f(x)^3.
\]
From this we deduce $f'(x) < (3/2)^{1/3} f(x) < 2f(x)$, as desired.

I don't know what the best constant is, except that it is not less than 1 (because $f(x) = e^x$ satisfies the given conditions).
 

Related to Can we prove that $f'(x)<2f(x)$ for all $x$ using the given conditions?

1. What are the given conditions for proving $f'(x)<2f(x)$ for all $x$?

The given conditions are that $f$ is a differentiable function and $f'(x)=2f(x)$ for at least one value of $x$.

2. How can we prove that $f'(x)<2f(x)$ for all $x$ based on the given conditions?

We can use the Mean Value Theorem, which states that for any differentiable function $f$ on an interval $[a,b]$, there exists a value $c$ in $(a,b)$ such that $f'(c)=\frac{f(b)-f(a)}{b-a}$. Therefore, if $f'(x)=2f(x)$ for at least one value of $x$, then for any other value of $x$ in the interval, $f'(c)<2f(c)$ must hold.

3. Are there any other methods for proving $f'(x)<2f(x)$ for all $x$?

Yes, we can also use proof by contradiction. Assume that there exists a value $x_0$ for which $f'(x_0)\geq2f(x_0)$. Then by definition of the derivative, we have $\lim_{x\to x_0}\frac{f(x)-f(x_0)}{x-x_0}\geq2f(x_0)$. This means that there exists a small interval around $x_0$ where the slope of the tangent line is always greater than or equal to $2f(x_0)$. However, this contradicts the given condition that $f'(x)=2f(x)$ for at least one value of $x$. Therefore, our assumption must be false and $f'(x)<2f(x)$ for all $x$.

4. Can we generalize this proof for any value greater than 2?

Yes, the same proof can be applied for any value greater than 2. If the condition is $f'(x)=kf(x)$ for some constant $k>2$, then the Mean Value Theorem and proof by contradiction can be used similarly to prove that $f'(x)

5. Is it possible for $f'(x)<2f(x)$ to hold for all $x$ without the given conditions?

No, without the given conditions, we cannot prove that $f'(x)<2f(x)$ for all $x$. The given conditions are necessary for proving the statement using the methods mentioned above. However, there may be other methods or approaches that can be used to prove the statement without the given conditions.

Similar threads

Replies
1
Views
1K
  • Math POTW for University Students
Replies
1
Views
1K
  • Math POTW for University Students
Replies
1
Views
2K
  • Math POTW for University Students
Replies
1
Views
1K
  • Math POTW for University Students
Replies
1
Views
2K
  • Math POTW for University Students
Replies
1
Views
1K
  • Math POTW for University Students
Replies
1
Views
1K
  • Math POTW for University Students
Replies
1
Views
2K
  • Math POTW for University Students
Replies
1
Views
2K
  • Math POTW for University Students
Replies
1
Views
2K
Back
Top